8
$\begingroup$

Let $A_n$ be the matrix whose rows and columns are indexed by pairs $(i,j)$ with $1\leq i,j\leq n$ and $i\neq j$ (so $A$ is an $n(n-1)\times n(n-1)$ matrix), whose $((i,j),(k,l))$-entry is 0 if $i=k$ or $j=l$, and is 1 otherwise. Evidence suggests that the characteristic polynomial of $A$ (normalized to be monic) is given by $$ \det(xI-A_n)=(x-1)^{n^2-3n+1}(x+n-1)^{n-1}(x+n-3)^{n-1}(x-n^2+3n-3). $$ Is this a known result? If not, how can it be proved?

$\endgroup$
2
  • 1
    $\begingroup$ If we dropped the $i \neq j$ restriction, then $A_n$ would be the Kronecker product $\left(E_n - I_n\right) \otimes \left(E_n - I_n\right)$, where $E_n$ is the $n\times n$ all-ones matrix. With the restriction, it is a diagonal block of this Kronecker product. Does this help? I don't know. $\endgroup$ Commented Dec 7, 2024 at 21:52
  • 1
    $\begingroup$ @darijgrinberg Yes, I think this helps. The cp of your matrix can easily be shown to be $(x-1)^{(n-1)^2} (x+n-1)^{2 n-2} (x-(n-1)^2)$. It remains to show that the determinant of the $n\times n$ Schur complement w.r.t. the rows & columns with equal pairs gives the missing factor $\frac{(x-1)^n (x+n-1)^{n-1} \left(x-(n-1)^2\right)}{(x+n-3)^{n-1}\left(x-(n-1)^2+n-2\right)}$. Mathematica says that this Schur complement is a rather simple linear combination of $E_n$ and $I_n$. $\endgroup$ Commented Dec 7, 2024 at 23:14

1 Answer 1

8
$\begingroup$

Let $E_m$ be the $m\times m$ all-ones matrix. We'll show that the spectrum of $$B_n:=I_{n(n-1)}+E_{n(n-1)}-A_n,$$ the matrix whose $((i,j),(k,\ell))$ entry is $1_{i=k}+1_{j=\ell}$, consists of $n^2-3n+1$ copies of zero, $n-1$ copies of $n-2$, $n-1$ copies of $n$, and one copy of $2n-2$. Since this one copy of $2n-2$ comes from the all-ones vector, it is easy to turn this into the desired description of the spectrum of $A_n$.

We do this by explicitly computing the eigenspaces. We will use the following description of the action of $B_n$ on a vector: $$(B_nv)_{ij}=\sum_{k\neq j}v_{kj}+\sum_{\ell\neq i}v_{i\ell}.\tag{$\star$}$$ Let $S=\{i,j\in[n]:i\neq j\}$. Let $\sigma:\mathbb R^S\to\mathbb R^S$ be the involution defined by $(\sigma v)_{ij}=v_{ji}$, define the subspace $$V=\left\{v\in\mathbb R^S:v_{ij}=v_{k\ell}\text{ whenever }i=k\right\};$$ we have that $\sigma V$ is the space of vectors $v$ for which $v_{ij}=v_{k\ell}$ whenever $j=\ell$. Note that $V\cap\sigma V$ consists only of the all-ones vector. As a result, the subspace $V+\sigma V$ has dimension $2\dim V-1=2n-1$. Property ($\star$) gives the description $$\frac1{n-1}B_n=\pi_V+\pi_{\sigma V},$$ where $\pi_W$ denotes the orthogonal projection onto a subspace $W$. From this, we learn the following:

  • On the subspace $V^\perp\cap(\sigma V)^\perp=(V+\sigma V)^\perp$ of $\mathbb R^S$, which has dimension $n(n-1)-(2n-1)=n^2-3n+1$, the map $B_n$ evaluates to zero.
  • On the subspace $V\cap\sigma V$, which is the span of the all-ones vector $\mathbf 1$, the map $B_n$ acts by $(2n-2)$ times the identity.
  • Let $W=(V+\sigma V)\cap\mathbf 1^\perp$. This space has dimension $2n-2$, and as the intersection of two subspaces of $\mathbb R^S$ invariant under both $\sigma$ and $B_n$, $W$ is invariant under $B_n$.

The first two of the above properties give us the eigenvalues of zero and $2n-2$, and the third tells us that the remaining eigenvalues will be found in $B_n|_W$. Since $\sigma^2=1$ and $\sigma$ is symmetric, the space $W$ splits as the direct sum of the two orthogonal pieces $W_\pm$, defined as the $\pm1$-eigenspaces of $\sigma|_W$. We claim that $W_+$ is an $(n-2)$-eigenspace of $B_n$ and that $W_-$ is an $n$-eigenspace of $B_n$.

Take any $w\in W_\pm$; we can write $w=v\pm\sigma v$ for some $v\in V\cap \mathbf 1^\perp$. Write $a_1,\ldots,a_n$ so that $v_{ij}=a_i$ for each $i$ and $j\neq i$; we have $a_1+\cdots+a_n=0$ and $w_{ij}=v_{ij}\pm v_{ji}=a_i\pm a_j$ for each $i\neq j$. So, by ($\star$), \begin{align*} (B_nw)_{ij} &=\sum_{k\neq j}w_{kj}+\sum_{\ell\neq i}w_{i\ell}\\ &=\sum_{k\neq j}(a_k\pm a_j)+\sum_{\ell\neq i}(a_i\pm a_\ell)\\ &=\left(\sum_{k\neq j}a_k\right)\pm(n-1)a_j+(n-1)a_i\pm \left(\sum_{\ell\neq i}a_\ell\right)\\ &=-a_j\pm(n-1)a_j+(n-1)a_i\mp a_i\\ &=(n-1\mp 1)(a_i\pm a_j)=(n-1\mp 1)w_{ij}, \end{align*} as desired.

$\endgroup$

You must log in to answer this question.

Start asking to get answers

Find the answer to your question by asking.

Ask question

Explore related questions

See similar questions with these tags.